Difference between revisions of "Aufgaben:Problem 5"

From Ferienserie MMP2
Jump to: navigation, search
(changed \hat f(k) to c_k to stress that these are fourier coefficients and not the fourier transform)
m (nitpicks)
 
(10 intermediate revisions by 6 users not shown)
Line 1: Line 1:
 +
Follow the link! (Original Solution Sheldon)
 +
[https://polybox.ethz.ch/public.php?service=files&t=3cd27159ac0fca5b53c53fe4cff98dcc]
  
=Problem=
+
An alternative solution can be found here (3.). (MuLö Linalg)
Show, by using the Fourier series, that
+
[http://www.math.ethz.ch/education/bachelor/lectures/hs2013/math/linalg1/l12]
$$\sum_{k=1}^\infty \frac{1}{k^n} = - \frac{(2 \pi i)^n}{2(n!)} B_n,\ n \in 2 \mathbb{Z}_{>0}$$
+
where \(B_n\) are the Bernoulli numbers defined as
+
$$\frac{z}{e^z - 1} = \sum_{n=0}^\infty B_n \frac{z^n}{n!}$$
+
  
''Hint:'' consider \( B_n(x) := \sum_{k \in \mathbb{Z} \setminus \{0\}} k^{-n} e^{ikx},\ n \in \mathbb{Z}_{\geq 2},\ x \in \mathbb{R} \)
+
== Task ==
 +
Determine the center of the algebra of complex d × d-matrices.
 +
Hint: the center of an algebra \(A\) is defined as \(Z(A) = \{\phi \in A | \phi \psi = \psi \phi \; \forall \psi \in A \} \).
  
 +
== Problem 5 (Sheldon) ==
 +
Let big bold letters denote elements of \(\mathbb{C}^{d \times d}\) and small Greek letters complex numbers.
  
=Solution Sketch=
+
Let \(M \in Z\)
The solution below is extremely comprehensive, if not expletive, because I felt that the [http://www.math.ethz.ch/~gruppe5/group5/group5.php?lectures/mmp/hs11/ml03.pdf Musterlösung] of the MMP Series that treated the problem was lacking some fairly important steps/guidelines. Therefore find a '''TL;DR''' here:
+
  
# See that \(B_n(0) = 2 \sum_{k=1}^\infty k^{-n}\). We now want to find the \(B_n(0)\) for \(n \in 2\mathbb{Z}_{>0}\)
+
We make the following observations and conclude that \( Z = \{\lambda\mathbb{I}\}\)
# We find a polynomial with Fourier coefficients \(c_0=0\) and \(c_k = \frac{1}{k^2}\). <br/> On \([0,2\pi]\), this polynomial will thus be equal to \(B_2(x)\)
+
# \(B'_n(x) = iB_{n-1}(x)\), therefore all \(B_n(x)\) will be polynomials, too. We find a recursion relation for \(B_n(x)\)
+
# Using the \(2\pi\)-periodicity of \(B_n(x)\) (for all \(n \geq 2\)), we find a recursion relation for the \(B_n(0)\)
+
# We prove the postulated identity by relating the definition for the \(B_n\) given in the problem to the recursion for the \(B_n(0)\) found above.
+
  
  
=Solution=
+
Obviously, the elementes of the center are closed under scalar multiplication (they even form a vector space over \(\mathbb{C}\)). \(\mathbb{I} \in Z \Rightarrow \{\lambda\mathbb{I}\} \subseteq Z\)
As suggested in the problem, we will have a look at the convergent fourier series
+
$$B_n(x) := \sum_{k \in \mathbb{Z} \setminus \{0\}} k^{-n} e^{ikx},\ n \in \mathbb{Z}_{\geq 2},\ x \in \mathbb{R}$$
+
with Fourier coefficients
+
$$c_k = \begin{cases}
+
0 & \text{for } k = 0\\
+
\frac{1}{k^n} & \text{for } k \neq 0
+
\end{cases}$$
+
  
  
It is obvious that \( B_n(x) \) is \(2\pi\)-periodic and, by differentiating every term separately, that
+
The theorem about Jordan decomposition states that \(M\) is conjugate to a matrix in Jordan normal form \(J\) and \(J = CMC^{-1}=MCC^{-1}=M\).
$$B'_n(x) = iB_{n-1}(x)$$
+
  
Also, for \(n \in 2\mathbb{Z}_{>0}\), we see that
+
\(M\) is a diagonal matrix.
$$B_n(0) = \sum_{k \in \mathbb{Z} \setminus \{0\}} k^{-n} e^{ik\,0}
+
= \sum_{k=-\infty}^{-1} k^{-n} + \sum_{k=1}^\infty k^{-n}
+
= 2 \sum_{k=1}^\infty k^{-n}
+
\tag{$\ddagger$}$$
+
where the last step follows from the fact that \(n\) is even.
+
  
<hr style="border:0;border-top:thin dashed #ccc;background:none;"/>
+
Suppose to the contrary that \(M\) has a '1' in the superdiagonal, then \(2M\) is supposed to be in the center as well, but it can't, because it is not in Jordan normal form.
  
We will first show that \( B_2(x) = f(x) := \pi^2/3 - \pi x + x^2/2 \)  for  \( x \in [0, 2\pi]\).
 
  
Recall the definition of the Fourier coefficients for \(2\pi\)-periodic functions in \(C^\infty(\mathbb{R})\):
+
All diagonal entries of \(M\) are equal.
$$c_k = \frac{1}{2\pi} \int_0^{2\pi} f(x) \: e^{-ikx} \: dx$$
+
  
We can easily verify the following identities:
+
Let \( M =\begin{pmatrix} \lambda_1 & 0 & \ldots & 0 \\ 0 & \lambda_2 & \ldots & 0 \\ \vdots & &\ddots& \\ 0 & \ldots & 0 & \lambda_d \end{pmatrix} \) and \( E_{ij} \) be the matrix with zero entries except for a '1' in the i-th row and j-th column.
$$\begin{align}
+
\frac{1}{2\pi} \int_0^{2\pi} e^{-ikx} \: dx &\ =\
+
\begin{cases}
+
1 & \text{for } k = 0\\
+
0 & \text{for } k \neq 0  
+
\end{cases} \\
+
\frac{1}{2\pi} \int_0^{2\pi} x \: e^{-ikx} \: dx &\ =\
+
\begin{cases}
+
\pi & \text{for } k = 0\\
+
\frac{i}{k} & \text{for } k \neq 0
+
\end{cases} \\
+
\frac{1}{2\pi} \int_0^{2\pi} x^2 \: e^{-ikx} \: dx &\ =\
+
\begin{cases}
+
\frac{4}{3} \pi^2 & \text{for } k = 0\\
+
\frac{2(1+ik\pi)}{k^2} & \text{for } k \neq 0
+
\end{cases}
+
\end{align}$$
+
  
Calculating the Fourier coefficients for \(f(x)\) then yields
+
\(\lambda_i E_{ij} = ME_{ij} = E_{ij}M = \lambda_j E_{ij} \Rightarrow \lambda_i = \lambda_j \)
$$\begin{align}
+
c_k &= \frac{1}{2\pi} \int_0^{2\pi} f(x) \: e^{-ikx} \: dx\\
+
&= \frac{1}{2\pi} \left(
+
\frac{\pi^2}{3} \int_0^{2\pi} e^{-ikx} \: dx
+
- \pi \int_0^{2\pi} x \: e^{-ikx} \: dx
+
+ \frac{1}{2} \int_0^{2\pi} x^2 \: e^{-ikx}
+
\right) \\
+
&= \begin{cases}
+
0 & \text{for } k = 0\\
+
\frac{1}{k^2} & \text{for } k \neq 0
+
\end{cases}
+
\end{align}$$
+
which proves that \(B_2(x) = f(x)\) for \(x \in [0, 2\pi]\).
+
  
Thus, we have shown that the \(B_n|_{[0, 2\pi]}\) are polynomials, which are recursively defined by \(B'_n = iB_{n-1}\) and \(\int_0^{2\pi} B_n(x) dx = -i(B_{n+1}(2\pi) - B_{n+1}(0)) = 0\). We also add
+
Because \( 1 \leq i \leq j \leq d \) can be chosen arbitrary, this proves the other inclusion \(M \in \{\lambda\mathbb{I}\}\).
$$B_1(x) = -iB'_2(x) = i\pi - ix \qquad B_0(x) = -iB'_1(x) = -1$$
+
  
<hr style="border:0;border-top:thin dashed #ccc;background:none;"/>
+
== Problem 5 (Musterloesung Illmanen) ==
 +
Let \(E_{ij} \in \mathbb{K}^{n\times n} \) be the matrix which has the entry 1 at \((i,j)\) and 0 else. From proposition \(AE_{ij} = E_{ij}A ~~ \forall 1 \leq i, j \leq n\). Let \(1 \leq i, j \leq n \) be fixed: \(AE_{ij} \) consists solely out of zeros except for the j-th column which is equal to \((a_{1i}, \ldots , a_{ni})^T\). So we get \( a_{ii} = a_{jj} ~~ \wedge  ~~ a_{ki} = 0 ~~\forall k \neq j\)
  
By induction over \(n\), we will now prove that
+
We can do this for all indices and this leads to \(a_{ij} = 0 \forall i \neq j ~~\wedge ~~ a_{11} = a_{22} = \ldots = a_{nn} \). Therefor \(A\) has to have the form \(\lambda E_n, \lambda \in \mathbb{K}\)
$$B_n(x) = \sum_{k=0}^n B_{n-k}(0) \frac{(ix)^k}{k!}$$
+
Base case for \(n=0\):
+
$$B_0(0) \frac{(ix)^0}{0!} = B_0(0) =  -1 = B_0(x)$$
+
Inductive step (\(n\!-\!1 \rightarrow n\)):
+
$$\begin{align}
+
B_n(x) &= \int_0^x iB_{n-1}(\tilde x)\ d\tilde x + B_n(0) \\
+
&= \int_0^x i\left(\sum_{k=0}^{n-1} B_{(n-1)-k}(0) \frac{(i\tilde x)^k}{k!} \right) d\tilde x + B_n(0) \\
+
&= \left( \sum_{k=0}^{n-1} B_{(n-1)-k}(0) \frac{i^{k+1}}{k!} \int_0^x \tilde x^k\ d\tilde x \right) + B_n(0) \\
+
&= \left( \sum_{k=0}^{n-1} B_{(n-1)-k}(0) \frac{i^{k+1}}{k!} \frac{x^{k+1}}{k+1} \right) + B_n(0) \\
+
&= \left( \sum_{k=0}^{n-1} B_{n-(k+1)}(0) \frac{(ix)^{k+1}}{(k+1)!} \right) + B_n(0) \\
+
&= \left( \sum_{k=1}^n B_{n-k}(0) \frac{(ix)^k}{k!} \right) + B_n(0) \\
+
&= \sum_{k=0}^n B_{n-k}(0) \frac{(ix)^k}{k!}
+
\end{align}$$
+
  
<hr style="border:0;border-top:thin dashed #ccc;background:none;"/>
+
===Ilmanen's solution componentwise===
  
As \(B_n\) is \(2\pi\)-periodic for all \(n \geq 2\), the above identity leads us to the relation
+
Let \(A \in Z(\mathrm{Mat}_d(\mathbb{C}))\).
$$0 = B_n(2\pi) - B_n(0) = \sum_{k=1}^n B_{n-k}(0) \frac{(2\pi i)^k}{k!} \tag{*}$$
+
for all \(n \geq 2\), which determines all \(B_n(0)\) from \(B_0(0) = -1\)
+
  
<hr style="border:0;border-top:thin dashed #ccc;background:none;"/>
+
Let \(E_{ij}\) be the \(d \times d\)-matrix with \((E_{ij})_{kl} = \delta_{ik} \delta_{jl} \).
  
The definition of the \(B_n\) in the problem (the Bernoulli numbers, not the functions!) can be transformed to:
+
Now let \(1 \leq i, j \leq d\) with \(i \neq j\) be fixed and consider
$$\begin{align}
+
$$(E_{ij} A)_{kl} = \sum \limits_{m}(E_{ij})_{km} A_{ml} = \sum \limits_{m}\delta_{ki} \delta_{jm} A_{ml} = \delta_{ki} A_{jl}.$$
& \frac{z}{e^z - 1} = \sum_{n=0}^\infty B_n \frac{z^n}{n!} \\
+
Since \(A\) commutes with all compex \(d \times d\)-matrices, this is the same as
\Leftrightarrow\ & z = \left( \sum_{n=0}^\infty B_n \frac{z^n}{n!} \right) \left( \sum_{k=1}^\infty \frac{z^k}{k!} \right) \\
+
$$(A E_{ij})_{kl} = \sum \limits_{m}A_{km} (E_{ij})_{ml} = \sum \limits_{m} A_{km} \delta_{mi} \delta_{jl} = \delta_{jl} A_{ki}.$$
\Leftrightarrow\ & z = \left( B_0 + B_1z + \frac{B_2}{2}z^2 + \frac{B_3}{3!}z^3 + ... \right) \left( z + \frac{z^2}{2} + \frac{z^3}{3!} + \frac{z^4}{4!} + ... \right) \\
+
\Leftrightarrow\ & z = B_0z + \left( \frac{B_0}{2!} + B_1 \right)z^2 + \left( \frac{B_0}{3!} + \frac{B_1}{2!} + \frac{B_2}{2!} \right)z^3 + \left( \frac{B_0}{4!} + \frac{B_1}{3!} + \frac{B_2}{2!2!} + \frac{B_3}{3!} \right)z^4
+
\end{align}$$
+
which, through equating coefficients yields \(B_0 = 1\) and for \(n \geq 2\)
+
$$\sum_{k=1}^n \frac{B_{n-k}}{(n-k)!k!} = 0 \tag{$\dagger$}$$
+
  
<hr style="border:0;border-top:thin dashed #ccc;background:none;"/>
+
Thus, we have that
 
+
$$\delta_{ki} A_{jl} = \delta_{jl} A_{ki}$$
<span style="color:#339;font-size:80%;">The following proof by induction doesn't appear in the aforementioned Musterlösung, where the final step simply falls out of thin air. It is not unlikely, however, that I simply don't understand what they do. If this is the case, feel free to simplify the following mess of equations. --Nik</span>
+
Now set \(k=i, l=j\). We then find
 
+
$$A_{ii} = A_{jj}$$
We now now prove by induction that for \(n \geq 0\)
+
Next, set \(k=l=i\). Still, \(j \neq i\). It follows that  
$$B_n(0) = - \frac{(2\pi i)^n B_n}{n!}$$
+
$$A_{ji} = 0.$$
 
+
As this holds for an arbitrary choice of \(i \neq j\), the required form for \(A\) is
Base case for \(n=0\):
+
$$A = \lambda \mathbb{I}_d$$
$$B_0(0) = -1 = -B_0$$
+
for some \(\lambda \in \mathbb{C}\). And obviously \(\forall \lambda \in \mathbb{C}: \lambda \mathbb{I} \in Z(\mathrm{Mat}_d(\mathbb{C}))\).
Inductive step (\(n\!-\!1 \rightarrow n\)):
+
$$\begin{align}
+
\sum_{k=1}^{n+1} B_{n+1-k}(0) \frac{(2\pi i)^k}{k!} \ &\overset{(*)}{=} \ 0 \\
+
\Leftrightarrow \ B_n(0)\: 2\pi i \ &= \: - \sum_{k=2}^{n+1} B_{n+1-k}(0) \frac{(2\pi i)^k}{k!} \\
+
\overset{\text{Hyp.}}{\Leftrightarrow} \ B_n(0)\: 2\pi i \ &= \: - \sum_{k=2}^{n+1} \left( - \frac{(2\pi i)^{n+1-k} B_{n+1-k}}{(n+1-k)!} \right) \frac{(2\pi i)^k}{k!} \\
+
\Leftrightarrow \ B_n(0)\: 2\pi i \ &= \ (2\pi i)^{n+1} \sum_{k=2}^{n+1} \frac{B_{n+1-k}}{(n+1-k)!\:k!} \\
+
\overset{(\dagger)}{\Leftrightarrow} \ B_n(0)\: 2\pi i \ &= \ (2\pi i)^{n+1} \left(- \frac{B_n}{n!} \right) \\
+
B_n(0) \ &= \: - \frac{(2\pi i)^n B_n}{n!}
+
\end{align}$$
+
 
+
This, together with equation \((\ddagger)\), proves the identity in the problem.
+
 
+
<p style="text-align:right;">\(\square\)</p>
+
 
+
 
+
 
+
 
+
=Alternate Solution=
+
<span style="color:#339;font-size:80%;">This solution has not yet been proof-read, but was proposed by Alessio (aka [[User:Milfhunter|Milfhunter]], veeery classy) as their TA solved the same problem in their tutor class.</span>
+
 
+
Let $$\zeta(s) = \sum\limits_{n=1}^\infty \frac{1}{n^s} $$ for $$s \in \mathbb{C}, \Re (s) > 1$$
+
 
+
Consider
+
$$ E(z) := \frac{z}{e^z -1} := \sum\limits_{n=0}^\infty B_n \frac{z^n}{n!} \quad\quad\quad z \in \mathbb{R} $$
+
$$\lim\limits_{z \to 0}{E(z)} = \lim\limits_{z \to 0}{\frac{1}{e^z}} = 1 = \lim\limits_{z \to 0}{\sum\limits_{n=0}^\infty B_n \frac{z^n}{n!}} $$
+
Where in the first equality we used l'Hospital's rule and with
+
$$ \lim\limits_{z \to 0}{E(z)} = E(\lim\limits_{z \to 0}{z}) = \sum\limits_{n=0}^\infty \lim\limits_{z \to 0}{B_n \frac{z^n}{n!}} =B_0 $$
+
Where we can put the limit inside because E is continuous and bounded in a neighborhood of z_0 = 0.
+
$$ \rightarrow B_0 = 1 $$
+
Now we repeat the same thing for E'
+
\begin{align*}
+
\lim\limits_{z \to 0}{E'(z)} & = \lim\limits_{z \to 0}{\frac{e^z -1 -z e^z}{e^{2z} -2e^z+1}} \\
+
& = \lim\limits_{z \to 0}{\frac{e^z -z e^z -e^z}{2e^{2z} -2e^z}} \\
+
& = \lim\limits_{z \to 0}{\frac{-z e^z - e^z}{4e^{2z}-2e^z}} \\
+
& = -\frac{1}{2}
+
\end{align*}
+
Here we again used l'Hospital's rule for the second and third equality. For the same reasons as above we get
+
$$ \lim\limits_{z \to 0}{E'(z)} = B_1 $$
+
and therefore
+
$$\rightarrow b_1 = - \frac{1}{2}$$
+
 
+
$$\Rightarrow E(z) = 1 - \frac{1}{2} z + \mathcal{O}(z^2)$$
+
 
+
Now define \(f(z)  = \frac{z}{2} \coth (\frac{z}{2})\) on \(]-\frac{1}{2}, \frac{1}{2}]\)
+
\begin{align*}
+
f(z) & = \frac{z}{2} \coth (\frac{z}{2}) \\
+
& = \frac{z e^{\frac{z}{2}} + z e^{-\frac{z}{2}}}{2e^{\frac{z}{2}} - 2 e^{-\frac{z}{2}}} \\
+
& = \frac{z e^{\frac{z}{2}} + z e^{-\frac{z}{2}}}{2e^{\frac{z}{2}}(e^z-1)} \\
+
& = \frac{z}{2} \frac{e^z+1}{e^z-1} \\
+
& = \frac{z}{2} \left( 1 + \frac{2}{e^z-1} \right) \\
+
& = \frac{z}{2} + \frac{z}{e^z-1} \\
+
& = \frac{z}{2} + E(z)
+
\end{align*}
+
 
+
Since z and coth are odd functions, f(z) is an even function. From the part above we know that $$E(z) = 1 - \frac{1}{2} z + \mathcal{O}(z^2)$$ Since the addition of z/2 kills the uneven term and f(z) has to be even, the only remaining coefficients are even:
+
$$ f(z) = \frac{z}{2} + E(z) = \sum\limits_{n=0}^\infty B_{2n} \frac{z^{2n}}{(2n)!} $$
+
 
+
Now define \(g(x) = \cosh (z x)\) on \(]-\frac{1}{2}, \frac{1}{2}]\)
+
with \(z \in \mathbb{R} \setminus \{0\}\), continued periodically on \(\mathbb{R}\)
+
Calculating the Fourier coefficients of g:
+
 
+
$$\alpha_0 = \int\limits_{-\frac{1}{2}} ^{\frac{1}{2}} \cosh(zx)dx = \frac{2}{z}\sinh (\frac{z}{2})$$
+
 
+
\begin{align*}
+
\alpha_n & = \int\limits_{-\frac{1}{2}} ^{\frac{1}{2}} \cosh(zx)\cos(2\pi n x)dx \\
+
& = \left[\cosh(zx)\frac{1}{2\pi n}\sin(2\pi nx) \right]_{-\frac{1}{2}}^{\frac{1}{2}} - \int\limits_{-\frac{1}{2}} ^{\frac{1}{2}} z \sinh (zx) \frac{1}{2\pi n} \sin (2\pi nx) dx \\
+
& = -z \int\limits_{-\frac{1}{2}} ^{\frac{1}{2}}\sinh (zx) \frac{1}{2\pi n} \sin (2\pi nx) dx \\
+
& = -z \left[ \left[-\sinh (zx) \left(\frac{1}{2\pi n}\right)^2 \cos (2\pi nx) \right]_{-\frac{1}{2}}^{\frac{1}{2}} + z \int\limits_{-\frac{1}{2}}^{\frac{1}{2}} \cosh (zx)\left(\frac{1}{2\pi n}\right)^2 \cos (2\pi nx) dx \right]
+
\end{align*}
+
 
+
$$ \Rightarrow \left(1 + \left(\frac{z}{2\pi n}\right)^2\right)\int\limits_{-\frac{1}{2}}^{\frac{1}{2}} \cosh (zx) \cos (2\pi nx) dx = 2z\left(\frac{z}{2\pi n}\right)^2 (-1)^n \sinh (\frac{z}{2}) $$
+
 
+
$$ \rightarrow \alpha_n = \int\limits_{-\frac{1}{2}}^{\frac{1}{2}} \cosh (zx) \cos (2\pi nx) dx = \frac{2z}{(2\pi n)^2 + z^2} (-1)^n \sinh (\frac{z}{2}) $$
+
Where we use $$\cos(2\pi n x)$$ because the integrand is even. Therefore we get for the Fourieseries of g:
+
 
+
$$ \cosh (zx)  = \frac{2}{z} \sinh (\frac{z}{2}) \left[ 1 + \sum\limits_{n \in \mathbb{Z} \\ n \neq 0} (-1)^n \frac{2z}{4{\pi}^2 n^2 + z^2} \cos (2\pi n x) \right] $$
+
$$ = \frac{2}{z} \sinh(\frac{z}{2}) \left[ 1 + 2\sum\limits_{n=1}^\infty (-1)^n \frac{2z}{4{\pi}^2 n^2 + z^2} \cos (2\pi n x) \right]$$
+
 
+
Now we chose $$x = \frac{1}{2}$$:
+
 
+
$$ \cosh (\frac{z}{2}) = \frac{2}{z} \sinh(\frac{z}{2}) \left[ 1 + 2\sum\limits_{n=1}^\infty (-1)^n \frac{z^2}{4\pi^2 n^2} \frac{1}{1+\frac{z^2}{4\pi^2 n^2}} \right] $$
+
 
+
$$ \underbrace{\frac{z}{2} \frac{\cosh (\frac{z}{2})}{\sinh(\frac{z}{2})}}_{f(z)}
+
= 1 + 2\sum\limits_{n=1}^\infty (-1)^n \frac{z^2}{4{\pi}^2 n^2} \frac{1}{1+\frac{z^2}{4{\pi}^2 n^2}} $$
+
$$ = 1 + \sum\limits_{n=1}^\infty \sum\limits_{k=0}^\infty \frac{2z^2}{4{\pi}^2 n^2} \left(-\frac{z^2}{4\pi^2 n^2}\right)^k$$
+
 
+
Because both series are absolute convergent and uniform convergent we get after some calculation:
+
$$ f(z) = 1 + \sum\limits_{k = 1}^\infty 2 (-1)^{k-1} \zeta(2k) \frac{z^{2k}}{(2\pi)^{2k}} $$
+
but above we also calculated
+
 
+
$$ f(z) =1 +  \sum\limits_{n=1}^\infty B_{2n} \frac{z^{2n}}{(2n)!} $$
+
With a coefficient comparison we get:
+
 
+
$$ \zeta(2n) = - \frac{B_{2n}(-1)^{2n}(2\pi)^{2n}}{2(2n)!} = -\frac{(2\pi i)^{2n}}{2(2n)!}B_{2n} \quad\quad\quad \forall n \in \mathbb{N} $$
+
which is what we had to show.
+

Latest revision as of 14:01, 30 July 2015

Follow the link! (Original Solution Sheldon) [1]

An alternative solution can be found here (3.). (MuLö Linalg) [2]

Task

Determine the center of the algebra of complex d × d-matrices. Hint: the center of an algebra \(A\) is defined as \(Z(A) = \{\phi \in A | \phi \psi = \psi \phi \; \forall \psi \in A \} \).

Problem 5 (Sheldon)

Let big bold letters denote elements of \(\mathbb{C}^{d \times d}\) and small Greek letters complex numbers.

Let \(M \in Z\)

We make the following observations and conclude that \( Z = \{\lambda\mathbb{I}\}\)


Obviously, the elementes of the center are closed under scalar multiplication (they even form a vector space over \(\mathbb{C}\)). \(\mathbb{I} \in Z \Rightarrow \{\lambda\mathbb{I}\} \subseteq Z\)


The theorem about Jordan decomposition states that \(M\) is conjugate to a matrix in Jordan normal form \(J\) and \(J = CMC^{-1}=MCC^{-1}=M\).

\(M\) is a diagonal matrix.

Suppose to the contrary that \(M\) has a '1' in the superdiagonal, then \(2M\) is supposed to be in the center as well, but it can't, because it is not in Jordan normal form.


All diagonal entries of \(M\) are equal.

Let \( M =\begin{pmatrix} \lambda_1 & 0 & \ldots & 0 \\ 0 & \lambda_2 & \ldots & 0 \\ \vdots & &\ddots& \\ 0 & \ldots & 0 & \lambda_d \end{pmatrix} \) and \( E_{ij} \) be the matrix with zero entries except for a '1' in the i-th row and j-th column.

\(\lambda_i E_{ij} = ME_{ij} = E_{ij}M = \lambda_j E_{ij} \Rightarrow \lambda_i = \lambda_j \)

Because \( 1 \leq i \leq j \leq d \) can be chosen arbitrary, this proves the other inclusion \(M \in \{\lambda\mathbb{I}\}\).

Problem 5 (Musterloesung Illmanen)

Let \(E_{ij} \in \mathbb{K}^{n\times n} \) be the matrix which has the entry 1 at \((i,j)\) and 0 else. From proposition \(AE_{ij} = E_{ij}A ~~ \forall 1 \leq i, j \leq n\). Let \(1 \leq i, j \leq n \) be fixed: \(AE_{ij} \) consists solely out of zeros except for the j-th column which is equal to \((a_{1i}, \ldots , a_{ni})^T\). So we get \( a_{ii} = a_{jj} ~~ \wedge ~~ a_{ki} = 0 ~~\forall k \neq j\)

We can do this for all indices and this leads to \(a_{ij} = 0 \forall i \neq j ~~\wedge ~~ a_{11} = a_{22} = \ldots = a_{nn} \). Therefor \(A\) has to have the form \(\lambda E_n, \lambda \in \mathbb{K}\)

Ilmanen's solution componentwise

Let \(A \in Z(\mathrm{Mat}_d(\mathbb{C}))\).

Let \(E_{ij}\) be the \(d \times d\)-matrix with \((E_{ij})_{kl} = \delta_{ik} \delta_{jl} \).

Now let \(1 \leq i, j \leq d\) with \(i \neq j\) be fixed and consider $$(E_{ij} A)_{kl} = \sum \limits_{m}(E_{ij})_{km} A_{ml} = \sum \limits_{m}\delta_{ki} \delta_{jm} A_{ml} = \delta_{ki} A_{jl}.$$ Since \(A\) commutes with all compex \(d \times d\)-matrices, this is the same as $$(A E_{ij})_{kl} = \sum \limits_{m}A_{km} (E_{ij})_{ml} = \sum \limits_{m} A_{km} \delta_{mi} \delta_{jl} = \delta_{jl} A_{ki}.$$

Thus, we have that $$\delta_{ki} A_{jl} = \delta_{jl} A_{ki}$$ Now set \(k=i, l=j\). We then find $$A_{ii} = A_{jj}$$ Next, set \(k=l=i\). Still, \(j \neq i\). It follows that $$A_{ji} = 0.$$ As this holds for an arbitrary choice of \(i \neq j\), the required form for \(A\) is $$A = \lambda \mathbb{I}_d$$ for some \(\lambda \in \mathbb{C}\). And obviously \(\forall \lambda \in \mathbb{C}: \lambda \mathbb{I} \in Z(\mathrm{Mat}_d(\mathbb{C}))\).